[Math] The inverse of Laplacian operator for different orders

ap.analysis-of-pdesfa.functional-analysisfourier analysislaplacian

I post this question in MSE couple of days before and get no response. So I repost it here for better luck. Thank you!


Let $u,v\in C_c^\infty(\Omega)$ and $\Omega\subset \mathbb R^N$ is open bounded with smooth boundary. Let $\Delta$ denote the Laplacian operator, $I$ denotes the identity operator and $t\in\mathbb R^+$ is a positive real number.

Let
$$
f(t):=\|(I-t\Delta)^{-1}\nabla u\|_{L^2}^2-\|(I-t\Delta)^{-\frac32}\nabla v\|_{L^2}^2.
$$
It is given that $\|\nabla u\|_{L^2}^2<\|\nabla v\|_{L^2}^2$, that is, $f(0)<0$; and I know that there exists $t_0>0$ such that $f(t_0)=0$. I also know that
$$
\int u = \int v.
$$

My question: Can I prove that $f(t)<0$ for $0<t<t_0$ and $f(t)>0$ for
$t>t_0$?


My try: I compute that
$$
\frac{d}{dt}(\|(I-t\Delta)^{-1}\nabla u\|_{L^2}^2)=-\|(I-t\Delta)^{-\frac32}\Delta u\|_{L^2}^2<0
$$
so I know $\|(I-t\Delta)^{-1} \nabla u\|_{L^2}^2$ is decreasing as $t$ increasing, so is $\|(I-t\Delta)^{-\frac32} \nabla v\|_{L^2}^2$. But I can't prove that the later one decreasing faster… I guess the order $3/2$ would do sth but I am not sure…


Also, I am wondering that how may I write

$$ \|(I-t\Delta)^{-s}u\|_{L^2}^2=\sum_{k=0}?? $$ where $t$ and $s$ are
real numbers, based on Fourier transform.

I was trying to look for Bessel potential but had no luck…

Please advise!

Best Answer

Disclaimer

The below argument applies directly to the previous version of the question. For the current version the basic argument still hold, using that $$ \| \nabla u\|_2 = \langle u, -\Delta u\rangle $$ and so behaves nicely for eigenfunctions of the Laplacian.

To make the argument also work for $C^\infty_c$ functions simply apply a cut-off near the boundary. This is a small perturbation and so you just need to add some small $\epsilon$ terms to the whole thing.

The added requirement that $\int u = \int v$ is of no consequence since the constructed example below has no zero-frequency component, and that the insertion of $\nabla$ in the new version of the question kills this component anyway.

End disclaimer

With the given information it seems unlikely.

Let $b,g,h$ be three eigenfunctions of $-\Delta$ with eigenvalues $\lambda_b \gg \lambda_g \gg \lambda_h$, normalized so $\|b\|_2 = \|g\|_2 = \|h\|_2 = 1$. Let $\tau_0 = 0$, $\tau_1 = 1/\lambda_g$, and $\tau_2 = 1 / \lambda_h$.

Let $u = a_g g$ and $v = b + a_h h$ where $a_g$ and $a_h$ are scalars. We have that $$ \begin{align} f(\tau_0) &= a_g^2 - (1 + a_h^2) \\ f(\tau_1) &= \frac{a_g^2}{4} - \frac{1}{(1 + \frac{\lambda_b}{\lambda_g})^3} - \frac{a_h^2}{(1 + \frac{\lambda_h}{\lambda_g})^3} > \frac{a_g^2}{4} - \frac{\lambda_g^3}{\lambda_b^3} - a_h^2 \\ f(\tau_2) &= - \frac{a_h^2}{8} - \frac{1}{(1 + \frac{\lambda_b}{\lambda_h})^3} + \frac{a_g^2}{(1 + \frac{\lambda_g}{\lambda_h})^2} < - \frac{a_h^2}{8} + \frac{a_g^2 \lambda_h^2}{\lambda_g^2} \end{align}$$ It is clear that for very large $t$, $f(t) > 0$ (since $t^{-1}$ decays slower than $t^{-3/2}$). So to produce an almost counterexample (only thing missing is that $u$ and $v$ are not in $C^\infty_0(\Omega)$), we just need to arrange

$$\begin{align} a_g^2 &< a_h^2 + 1 \\ a_g^2 & \geq 4 a_h^2 + \frac{4 \lambda_g^3}{\lambda_b^3} \\ a_g^2 & \leq \frac{\lambda_g^2}{8 \lambda_h^2} a_h^2 \end{align} $$

This can be satisfied by, for example, $a_h^2 = 1/6$ and $a_g^2 = 1$ provided the ratio of eigenvalues $24\lambda_g^3 < \lambda_b^3$ and $48 \lambda_h^2 < \lambda_g^2$.

Related Question